Kwadrat liczby całkowitej

Podzielność. Reszty z dzielenia. Kongruencje. Systemy pozycyjne. Równania diofantyczne. Liczby pierwsze i względnie pierwsze. NWW i NWD.
kubek1
Użytkownik
Użytkownik
Posty: 249
Rejestracja: 15 wrz 2008, o 19:35
Płeć: Mężczyzna
Lokalizacja: Syberia
Podziękował: 15 razy
Pomógł: 32 razy

Kwadrat liczby całkowitej

Post autor: kubek1 »

Witam. Mam problem z następującym zadankiem:

Niech a i b będą takimi liczbami naturalnymi, że \(\displaystyle{ a|b+1}\) i \(\displaystyle{ b|a^2-2}\). Udowodnij, że liczba \(\displaystyle{ \frac{b+1}{2}}\) jest kwadratem pewnej liczby całkowitej.

Z góry dzięki za pomoc
tometomek91
Użytkownik
Użytkownik
Posty: 2959
Rejestracja: 8 sie 2009, o 23:05
Płeć: Mężczyzna
Lokalizacja: Wrocław
Podziękował: 281 razy
Pomógł: 498 razy

Kwadrat liczby całkowitej

Post autor: tometomek91 »

Z pierwszego warunku wynika, że albo liczby \(\displaystyle{ a}\) i \(\displaystyle{ b}\) są przeciwnej parzystości, albo a=1. Z drugiego natomiast wynika że obydwie są parzyste lub obydwie nieparzyste. Stąd \(\displaystyle{ a=b=1}\) i mamy tezę.
Awatar użytkownika
smigol
Użytkownik
Użytkownik
Posty: 3454
Rejestracja: 20 paź 2007, o 23:10
Płeć: Mężczyzna
Lokalizacja: Warszawa
Podziękował: 89 razy
Pomógł: 353 razy

Kwadrat liczby całkowitej

Post autor: smigol »

Z drugiego natomiast wynika że obydwie są parzyste lub obydwie nieparzyste.
a=6,b=17.

-- 28 sierpnia 2010, 19:54 --
Z pierwszego warunku wynika, że albo liczby i są przeciwnej parzystości, albo a=1.
b=11, a=3.
Awatar użytkownika
SaxoN
Użytkownik
Użytkownik
Posty: 154
Rejestracja: 20 cze 2008, o 14:33
Płeć: Mężczyzna
Lokalizacja: Katowice/ Warszawa
Podziękował: 3 razy
Pomógł: 9 razy

Kwadrat liczby całkowitej

Post autor: SaxoN »

Dla \(\displaystyle{ a\in\{1,2\}}\) zadanie jest trywialne. Załóżmy zatem \(\displaystyle{ a\geq 3}\) Z pierwszej podzielności \(\displaystyle{ b=aq-1}\), przy czym oczywiście \(\displaystyle{ (b,q)=1}\) oraz \(\displaystyle{ q=\frac{b+1}{a}\leq\frac{b+1}{3}\leq\frac{b}{2}}\), czyli \(\displaystyle{ 2q\leq b}\). Dalej \(\displaystyle{ b\mid \Big(\frac{b+1}{q}\Big)^2-2=\frac{b^2+2b+1-2q^2}{q^2}}\), czyli \(\displaystyle{ b\mid 2q^2-1=aq-1+q(2q-a)=b-q(2q-a)}\), a ponieważ \(\displaystyle{ b, q}\) są względnie pierwsze, \(\displaystyle{ b|2q-a\leq b-a< b}\), zatem \(\displaystyle{ 2q-a=0}\). To już kończy dowód, bo \(\displaystyle{ b=aq-1=2q^2-1\Rightarrow \frac{b+1}{2}=q^2}\).
kubek1
Użytkownik
Użytkownik
Posty: 249
Rejestracja: 15 wrz 2008, o 19:35
Płeć: Mężczyzna
Lokalizacja: Syberia
Podziękował: 15 razy
Pomógł: 32 razy

Kwadrat liczby całkowitej

Post autor: kubek1 »

Dzięki
ODPOWIEDZ